Showing that the Dirichlet function has no limit












2














Let $f(x) = 0$ if $x$ is rational and $f(x)=1$ if $x$ is irrational. Prove that $lim_{xto a} f(x)$ does not exist for any $a$.



Proof attempt:



Let $ainmathbb{R}$.
The only two possible values for a limit of $lim_{xto a}f(x)$ are $0,1$, since otherwise there will always be a constant gap between the valules of $f$ and the limit, and hence $f$ would not get arbitrarily small.



However, $lim_{xto a}f(x) not = 1$ for the following reason: Set $epsilon = 1/2$ and let $delta > 0$. Then there exists a rational number $x_0$ in $0<|x_0-a|<delta$ by the density of the rationals. However, $|f(x_0)-1| = |0-1| = 1 not < 1/2$, and so $lim_{xto a}f(x) not = 1$.



Also, $lim_{xto a}f(x) not = 0$ for the following reason: Set $epsilon = 1/2$ and let $delta > 0$. Then there exists a irrational number $x_0$ in $0<|x_0-a|<delta$ by the density of the irrationals. However, $|f(x_0)-0| = |1-0| = 1 not < 1/2$, and so $lim_{xto a}f(x) not = 0$.










share|cite|improve this question






















  • Exactly what are you asking? What point is the one you are unsure about?
    – Will M.
    Nov 21 '18 at 4:13










  • I am unsure of whether my proof is correct. For example, is assuming that the only possible limits are 0 and 1 a correct assumption? Then do I correctly rule these out as limits?
    – Wesley
    Nov 21 '18 at 4:15












  • Ok, technically speaking, you need to consider a value, whatever it may be, $L in mathbf{R},$ and then rule it out as a possible limit. You can divide three cases: (1) $L neq 0, 1$ (2) $L = 0$ and (3) $L=1.$
    – Will M.
    Nov 21 '18 at 4:17










  • Is at least my reasoning for cases (2) and (3) correct?
    – Wesley
    Nov 21 '18 at 4:30
















2














Let $f(x) = 0$ if $x$ is rational and $f(x)=1$ if $x$ is irrational. Prove that $lim_{xto a} f(x)$ does not exist for any $a$.



Proof attempt:



Let $ainmathbb{R}$.
The only two possible values for a limit of $lim_{xto a}f(x)$ are $0,1$, since otherwise there will always be a constant gap between the valules of $f$ and the limit, and hence $f$ would not get arbitrarily small.



However, $lim_{xto a}f(x) not = 1$ for the following reason: Set $epsilon = 1/2$ and let $delta > 0$. Then there exists a rational number $x_0$ in $0<|x_0-a|<delta$ by the density of the rationals. However, $|f(x_0)-1| = |0-1| = 1 not < 1/2$, and so $lim_{xto a}f(x) not = 1$.



Also, $lim_{xto a}f(x) not = 0$ for the following reason: Set $epsilon = 1/2$ and let $delta > 0$. Then there exists a irrational number $x_0$ in $0<|x_0-a|<delta$ by the density of the irrationals. However, $|f(x_0)-0| = |1-0| = 1 not < 1/2$, and so $lim_{xto a}f(x) not = 0$.










share|cite|improve this question






















  • Exactly what are you asking? What point is the one you are unsure about?
    – Will M.
    Nov 21 '18 at 4:13










  • I am unsure of whether my proof is correct. For example, is assuming that the only possible limits are 0 and 1 a correct assumption? Then do I correctly rule these out as limits?
    – Wesley
    Nov 21 '18 at 4:15












  • Ok, technically speaking, you need to consider a value, whatever it may be, $L in mathbf{R},$ and then rule it out as a possible limit. You can divide three cases: (1) $L neq 0, 1$ (2) $L = 0$ and (3) $L=1.$
    – Will M.
    Nov 21 '18 at 4:17










  • Is at least my reasoning for cases (2) and (3) correct?
    – Wesley
    Nov 21 '18 at 4:30














2












2








2


1





Let $f(x) = 0$ if $x$ is rational and $f(x)=1$ if $x$ is irrational. Prove that $lim_{xto a} f(x)$ does not exist for any $a$.



Proof attempt:



Let $ainmathbb{R}$.
The only two possible values for a limit of $lim_{xto a}f(x)$ are $0,1$, since otherwise there will always be a constant gap between the valules of $f$ and the limit, and hence $f$ would not get arbitrarily small.



However, $lim_{xto a}f(x) not = 1$ for the following reason: Set $epsilon = 1/2$ and let $delta > 0$. Then there exists a rational number $x_0$ in $0<|x_0-a|<delta$ by the density of the rationals. However, $|f(x_0)-1| = |0-1| = 1 not < 1/2$, and so $lim_{xto a}f(x) not = 1$.



Also, $lim_{xto a}f(x) not = 0$ for the following reason: Set $epsilon = 1/2$ and let $delta > 0$. Then there exists a irrational number $x_0$ in $0<|x_0-a|<delta$ by the density of the irrationals. However, $|f(x_0)-0| = |1-0| = 1 not < 1/2$, and so $lim_{xto a}f(x) not = 0$.










share|cite|improve this question













Let $f(x) = 0$ if $x$ is rational and $f(x)=1$ if $x$ is irrational. Prove that $lim_{xto a} f(x)$ does not exist for any $a$.



Proof attempt:



Let $ainmathbb{R}$.
The only two possible values for a limit of $lim_{xto a}f(x)$ are $0,1$, since otherwise there will always be a constant gap between the valules of $f$ and the limit, and hence $f$ would not get arbitrarily small.



However, $lim_{xto a}f(x) not = 1$ for the following reason: Set $epsilon = 1/2$ and let $delta > 0$. Then there exists a rational number $x_0$ in $0<|x_0-a|<delta$ by the density of the rationals. However, $|f(x_0)-1| = |0-1| = 1 not < 1/2$, and so $lim_{xto a}f(x) not = 1$.



Also, $lim_{xto a}f(x) not = 0$ for the following reason: Set $epsilon = 1/2$ and let $delta > 0$. Then there exists a irrational number $x_0$ in $0<|x_0-a|<delta$ by the density of the irrationals. However, $|f(x_0)-0| = |1-0| = 1 not < 1/2$, and so $lim_{xto a}f(x) not = 0$.







real-analysis limits proof-verification






share|cite|improve this question













share|cite|improve this question











share|cite|improve this question




share|cite|improve this question










asked Nov 21 '18 at 4:11









Wesley

525313




525313












  • Exactly what are you asking? What point is the one you are unsure about?
    – Will M.
    Nov 21 '18 at 4:13










  • I am unsure of whether my proof is correct. For example, is assuming that the only possible limits are 0 and 1 a correct assumption? Then do I correctly rule these out as limits?
    – Wesley
    Nov 21 '18 at 4:15












  • Ok, technically speaking, you need to consider a value, whatever it may be, $L in mathbf{R},$ and then rule it out as a possible limit. You can divide three cases: (1) $L neq 0, 1$ (2) $L = 0$ and (3) $L=1.$
    – Will M.
    Nov 21 '18 at 4:17










  • Is at least my reasoning for cases (2) and (3) correct?
    – Wesley
    Nov 21 '18 at 4:30


















  • Exactly what are you asking? What point is the one you are unsure about?
    – Will M.
    Nov 21 '18 at 4:13










  • I am unsure of whether my proof is correct. For example, is assuming that the only possible limits are 0 and 1 a correct assumption? Then do I correctly rule these out as limits?
    – Wesley
    Nov 21 '18 at 4:15












  • Ok, technically speaking, you need to consider a value, whatever it may be, $L in mathbf{R},$ and then rule it out as a possible limit. You can divide three cases: (1) $L neq 0, 1$ (2) $L = 0$ and (3) $L=1.$
    – Will M.
    Nov 21 '18 at 4:17










  • Is at least my reasoning for cases (2) and (3) correct?
    – Wesley
    Nov 21 '18 at 4:30
















Exactly what are you asking? What point is the one you are unsure about?
– Will M.
Nov 21 '18 at 4:13




Exactly what are you asking? What point is the one you are unsure about?
– Will M.
Nov 21 '18 at 4:13












I am unsure of whether my proof is correct. For example, is assuming that the only possible limits are 0 and 1 a correct assumption? Then do I correctly rule these out as limits?
– Wesley
Nov 21 '18 at 4:15






I am unsure of whether my proof is correct. For example, is assuming that the only possible limits are 0 and 1 a correct assumption? Then do I correctly rule these out as limits?
– Wesley
Nov 21 '18 at 4:15














Ok, technically speaking, you need to consider a value, whatever it may be, $L in mathbf{R},$ and then rule it out as a possible limit. You can divide three cases: (1) $L neq 0, 1$ (2) $L = 0$ and (3) $L=1.$
– Will M.
Nov 21 '18 at 4:17




Ok, technically speaking, you need to consider a value, whatever it may be, $L in mathbf{R},$ and then rule it out as a possible limit. You can divide three cases: (1) $L neq 0, 1$ (2) $L = 0$ and (3) $L=1.$
– Will M.
Nov 21 '18 at 4:17












Is at least my reasoning for cases (2) and (3) correct?
– Wesley
Nov 21 '18 at 4:30




Is at least my reasoning for cases (2) and (3) correct?
– Wesley
Nov 21 '18 at 4:30










1 Answer
1






active

oldest

votes


















3














Your proof is complete and fine. Only one minor correction : "hence $f$ would not get arbitrarily small" should be changed to "hence $f$ would not get arbitrarily close to the possible limit", in the first paragraph of the proof (or changed to "hence $f-L$ would not get arbitrarily small, where $L neq 0,1$ is a possible limit").



There is another proof that will avoid the breaking of $L$ into cases : indeed, given $a in mathbb R$, by density of rationals and irrationals, there is a sequence of rationals and irrationals converging to $a$. But, $lim_{x to a} f(x)$ exists if and only if for any two sequences $a_n, b_n to a$ we have $lim a_n = lim b_n$, and here the sequence of rationals and irrationals produce different values, namely $0$ and $1$.






share|cite|improve this answer





















    Your Answer





    StackExchange.ifUsing("editor", function () {
    return StackExchange.using("mathjaxEditing", function () {
    StackExchange.MarkdownEditor.creationCallbacks.add(function (editor, postfix) {
    StackExchange.mathjaxEditing.prepareWmdForMathJax(editor, postfix, [["$", "$"], ["\\(","\\)"]]);
    });
    });
    }, "mathjax-editing");

    StackExchange.ready(function() {
    var channelOptions = {
    tags: "".split(" "),
    id: "69"
    };
    initTagRenderer("".split(" "), "".split(" "), channelOptions);

    StackExchange.using("externalEditor", function() {
    // Have to fire editor after snippets, if snippets enabled
    if (StackExchange.settings.snippets.snippetsEnabled) {
    StackExchange.using("snippets", function() {
    createEditor();
    });
    }
    else {
    createEditor();
    }
    });

    function createEditor() {
    StackExchange.prepareEditor({
    heartbeatType: 'answer',
    autoActivateHeartbeat: false,
    convertImagesToLinks: true,
    noModals: true,
    showLowRepImageUploadWarning: true,
    reputationToPostImages: 10,
    bindNavPrevention: true,
    postfix: "",
    imageUploader: {
    brandingHtml: "Powered by u003ca class="icon-imgur-white" href="https://imgur.com/"u003eu003c/au003e",
    contentPolicyHtml: "User contributions licensed under u003ca href="https://creativecommons.org/licenses/by-sa/3.0/"u003ecc by-sa 3.0 with attribution requiredu003c/au003e u003ca href="https://stackoverflow.com/legal/content-policy"u003e(content policy)u003c/au003e",
    allowUrls: true
    },
    noCode: true, onDemand: true,
    discardSelector: ".discard-answer"
    ,immediatelyShowMarkdownHelp:true
    });


    }
    });














    draft saved

    draft discarded


















    StackExchange.ready(
    function () {
    StackExchange.openid.initPostLogin('.new-post-login', 'https%3a%2f%2fmath.stackexchange.com%2fquestions%2f3007254%2fshowing-that-the-dirichlet-function-has-no-limit%23new-answer', 'question_page');
    }
    );

    Post as a guest















    Required, but never shown

























    1 Answer
    1






    active

    oldest

    votes








    1 Answer
    1






    active

    oldest

    votes









    active

    oldest

    votes






    active

    oldest

    votes









    3














    Your proof is complete and fine. Only one minor correction : "hence $f$ would not get arbitrarily small" should be changed to "hence $f$ would not get arbitrarily close to the possible limit", in the first paragraph of the proof (or changed to "hence $f-L$ would not get arbitrarily small, where $L neq 0,1$ is a possible limit").



    There is another proof that will avoid the breaking of $L$ into cases : indeed, given $a in mathbb R$, by density of rationals and irrationals, there is a sequence of rationals and irrationals converging to $a$. But, $lim_{x to a} f(x)$ exists if and only if for any two sequences $a_n, b_n to a$ we have $lim a_n = lim b_n$, and here the sequence of rationals and irrationals produce different values, namely $0$ and $1$.






    share|cite|improve this answer


























      3














      Your proof is complete and fine. Only one minor correction : "hence $f$ would not get arbitrarily small" should be changed to "hence $f$ would not get arbitrarily close to the possible limit", in the first paragraph of the proof (or changed to "hence $f-L$ would not get arbitrarily small, where $L neq 0,1$ is a possible limit").



      There is another proof that will avoid the breaking of $L$ into cases : indeed, given $a in mathbb R$, by density of rationals and irrationals, there is a sequence of rationals and irrationals converging to $a$. But, $lim_{x to a} f(x)$ exists if and only if for any two sequences $a_n, b_n to a$ we have $lim a_n = lim b_n$, and here the sequence of rationals and irrationals produce different values, namely $0$ and $1$.






      share|cite|improve this answer
























        3












        3








        3






        Your proof is complete and fine. Only one minor correction : "hence $f$ would not get arbitrarily small" should be changed to "hence $f$ would not get arbitrarily close to the possible limit", in the first paragraph of the proof (or changed to "hence $f-L$ would not get arbitrarily small, where $L neq 0,1$ is a possible limit").



        There is another proof that will avoid the breaking of $L$ into cases : indeed, given $a in mathbb R$, by density of rationals and irrationals, there is a sequence of rationals and irrationals converging to $a$. But, $lim_{x to a} f(x)$ exists if and only if for any two sequences $a_n, b_n to a$ we have $lim a_n = lim b_n$, and here the sequence of rationals and irrationals produce different values, namely $0$ and $1$.






        share|cite|improve this answer












        Your proof is complete and fine. Only one minor correction : "hence $f$ would not get arbitrarily small" should be changed to "hence $f$ would not get arbitrarily close to the possible limit", in the first paragraph of the proof (or changed to "hence $f-L$ would not get arbitrarily small, where $L neq 0,1$ is a possible limit").



        There is another proof that will avoid the breaking of $L$ into cases : indeed, given $a in mathbb R$, by density of rationals and irrationals, there is a sequence of rationals and irrationals converging to $a$. But, $lim_{x to a} f(x)$ exists if and only if for any two sequences $a_n, b_n to a$ we have $lim a_n = lim b_n$, and here the sequence of rationals and irrationals produce different values, namely $0$ and $1$.







        share|cite|improve this answer












        share|cite|improve this answer



        share|cite|improve this answer










        answered Nov 21 '18 at 5:02









        астон вілла олоф мэллбэрг

        37.3k33376




        37.3k33376






























            draft saved

            draft discarded




















































            Thanks for contributing an answer to Mathematics Stack Exchange!


            • Please be sure to answer the question. Provide details and share your research!

            But avoid



            • Asking for help, clarification, or responding to other answers.

            • Making statements based on opinion; back them up with references or personal experience.


            Use MathJax to format equations. MathJax reference.


            To learn more, see our tips on writing great answers.





            Some of your past answers have not been well-received, and you're in danger of being blocked from answering.


            Please pay close attention to the following guidance:


            • Please be sure to answer the question. Provide details and share your research!

            But avoid



            • Asking for help, clarification, or responding to other answers.

            • Making statements based on opinion; back them up with references or personal experience.


            To learn more, see our tips on writing great answers.




            draft saved


            draft discarded














            StackExchange.ready(
            function () {
            StackExchange.openid.initPostLogin('.new-post-login', 'https%3a%2f%2fmath.stackexchange.com%2fquestions%2f3007254%2fshowing-that-the-dirichlet-function-has-no-limit%23new-answer', 'question_page');
            }
            );

            Post as a guest















            Required, but never shown





















































            Required, but never shown














            Required, but never shown












            Required, but never shown







            Required, but never shown

































            Required, but never shown














            Required, but never shown












            Required, but never shown







            Required, but never shown







            Popular posts from this blog

            Can a sorcerer learn a 5th-level spell early by creating spell slots using the Font of Magic feature?

            Does disintegrating a polymorphed enemy still kill it after the 2018 errata?

            A Topological Invariant for $pi_3(U(n))$